LSAT and Law School Admissions Forum

Get expert LSAT preparation and law school admissions advice from PowerScore Test Preparation.

 Administrator
PowerScore Staff
  • PowerScore Staff
  • Posts: 8917
  • Joined: Feb 02, 2011
|
#24017
Complete Question Explanation

Parallel Reasoning—SN. The correct answer choice is (C)

The first sentence of the stimulus is a conditional statement as well as the premise upon which the conclusion is based. The second sentence, also a conditional statement, is the conclusion. We can diagram the first statement as follows:
  • CCMS ..... :arrow: ..... 2%

(if the city council maintains spending, then 2% sales tax expected). The contrapositive of this statement would then hold that if the sales tax is not 2%, then it is likely the council did not maintain spending levels:
  • 2% ..... :arrow: ..... CCMS.


The conclusion then states “if the council levies a higher tax, it will be because the council is increasing expenditures,” a statement diagrammed as
  • ↑Tax ..... :arrow: ..... ↑Expenditures.


This flawed reasoning assumes that, because an unchanged amount of spending will likely result in a 2% sales tax, an increase in that sales tax figure automatically indicates an increase in the amount of spending. To parallel this flaw, look for an answer that similarly confuses the sufficient and necessary conditions of the premise while also mistakenly taking liberties with increase/decrease/maintain relationships stemming from that premise.

Answer Choice (A): The original stimulus utilizes the same conditions (tax rate and expenditure levels) in its premise as well as its conclusion. Unlike the original argument, this answer choice introduces an entirely new condition (enabling builders to sell a greater number of houses) not contained in the initial premise and can not be correct as a result.

Answer Choice (B): As with Answer Choice (A) , this answer does not confuse the sufficient and necessary conditions of the premise, but rather introduces a new condition (reduced profits) and is incorrect as a result.

Answer Choice (C):This is the correct answer choice.
The answer choice parallels the stimulus in that the conditions present in the premise (workers’ wages and the price of goods) are also present in the conclusion. This answer choice further parallels the original stimulus in that it confuses sufficient and necessary conditions while also making unwarranted assumptions regarding the relationship of increases/decreases based on the premise.

Answer Choice (D): As is the case with previous incorrect answer choices, this answer choice does not confuse sufficient and necessary conditions as its conclusion does not contain the same conditions as the premise upon which it is based. Here, the idea of improving service serves as the new condition not present in the premise and renders the answer incorrect.

Answer Choice (E): Once again an answer choice, rather than confusing sufficient and necessary conditions reaches a conclusion with a condition not contained in the premise (falling circulation in this example) and is incorrect as a result.
 sgrimsdale
  • Posts: 14
  • Joined: Sep 01, 2014
|
#18814
Hi,

The explanation of the answer confused me as it described the reasoning used as flawed. I was under the impression that the reasoning used was not flawed. I diagrammed the stimulus as: Maintain Spending ----> 2% Tax. The contrapositive would therefore be, Not 2% Tax ------> Not Maintain Spending. I understood the conclusion to merely restate the contrapositive as Levying a Higher Tax = Not 2% Tax, and Increasing Expenditures = Not Maintaining Spending. Additionally, I thought that Parallel Flaw questions always identified the reasoning as flawed? I'm clearly missing something here. Can someone explain where I went wrong?

Thank you!

Sarah
 Steve Stein
PowerScore Staff
  • PowerScore Staff
  • Posts: 1153
  • Joined: Apr 11, 2011
|
#18819
Hi Sarah,

That's a great question, and you are correct. That argument isn't really flawed--one way we might diagram it would be to diagram the condition "maintaining spending" as NOT increasing spending:

NOT increase spending :arrow: 2% tax
NOT 2% tax (its higher) :arrow: Increase spending.

Similarly, correct answer choice (C) provides for the following diagram:

NOT increase wages :arrow: same prices as last year
NOT same prices (they're higher) :arrow: increaase wages

The explanation for that question will be amended and expanded. Thanks again for letting us know, and please let us know if we can answer any other questions about the LSAT or about your preparation in general.

Thanks!

Steve
 Johnclem
  • Posts: 122
  • Joined: Dec 31, 2015
|
#26603
Hi powerscore,
I seem to be disagreeing with the explanations for this question. I do believe the argument is flawed but not because of conditionality but rather causality wise. Here is my reasoning . What am I missing ?

1) maintains spending --> 2% tax
2) NOT 2% tax --> Spending NOT maintained ( this is translated from the premises : " if higher tax , it will be because the council is increasing expenditures" )

Now the flaw :
The author concludes using causality. He /she states that if we have higher tax it is because the council is increasing its spending . ( Increasing spending --- higher tax ) this is flawed because there could be an alternate reason for the higher tax.


Thanks
John
 Clay Cooper
PowerScore Staff
  • PowerScore Staff
  • Posts: 241
  • Joined: Jul 03, 2015
|
#26608
Hi Johnclem,

Thanks for your question.

Reading back through these posts, it sounds to me like the jury is still out (lol, law school joke) on whether this stimulus actually contains a flaw or not. I think it does not - but if it does, you have spotted it. Here is how I have diagrammed the stimulus:

~Increased spending :arrow: 2% tax
~2% tax :arrow: increased spending

and the correct answer choice:

~increase wages :arrow: ~prices change
prices change :arrow: increase wages

Now, obviously, in each case the second statement is the contrapositive of the first, so we have not made any straightforward errors of conditional reasoning. I think, instead, that if an error exists, it is (as the explanation points out) the assumption each argument makes: that raised taxes can only be caused by increased expenditures (and not decreased expenditures); and that increased prices can only be caused by increased wages.

However, after some thought and confusion, I don't think these leaps are errors; I think that common sense allows us make the leap that - if we know unchanged spending means unchanged taxes - increased taxes must mean increased spending.

I think, at the end of the day, whether or not an error is present simplifies to this question: is that particular logical step sound? It exists both in the stimulus and in the correct answer choice.

I think, more importantly, that it doesn't matter; whether or not you or I personally think the argument is valid, the correct answer choice undoubtedly is the most parallel to the stimulus.

This was fun! Keep working hard.

Get the most out of your LSAT Prep Plus subscription.

Analyze and track your performance with our Testing and Analytics Package.